The perimeter of a patio is 64 feet. The width of the patio is 12 feet and the length of the patio is ( ) x + 6 feet. What is the length of the patio?
What is the answer to this question?
Answer:
≈ 3.9 miles
Step-by-step explanation:
The third angle in the triangle is
180° - (53 + 78)° = 180° - 131° = 49°
Using the Sine rule in the triangle with distance from 2 being x, then
\(\frac{3}{sin49}\) = \(\frac{x}{sin78}\) ( cross- multiply )
x × sin49° = 3 × sin78° ( divide both sides by sin49° )
x = \(\frac{3sin78}{sin49}\) ≈ 3.9 ( to the nearest tenth )
In △XYZ, m∠X = 27° and m∠Y = 48°. Which gives the sides of the triangle in order from shortest to longest?
Answer:
z, y and x
Step-by-step explanation:
Now, to answer this question, what we must know is that the side facing the largest angle is the largest while the side facing the smallest is the smallest.
What i’m saying in essence is that the bigger the angle faced, the larger the side.
Now since we have 3 angles in a triangle that adds up to 180
we have 27 and 48 already, the third side would be 180-27-48 = 105
So which side faces which angle?
Let’s have a diagrammatic representation of the said triangle.
Check this in the attachment please
The longest side is thus z, followed by y and lastly x
The measure of the angle from shortest to longest will be ∠z, ∠y, and ∠x.
What is angle measurement?An angle measure is the measurement of the angle created by two rays or arms at a shared vertex in geometry. A protractor is used to measure angles in degrees (°).
Given data;
m∠X = 27°
m∠Y = 48°
The sum of the angle of the triangle is 180°
m∠X+m∠Y+m∠Z = 180°
27° +48° + m∠Z = 180°
m∠Z = 105°
The order of the angle measurement;
m∠Z > m∠Y > m∠X
Hence, the measure of the angle from shortest to longest will be ∠z, ∠y, and ∠x.
To learn more about the angle measurement, refer to the link;
https://brainly.com/question/14684647
#SPJ5
What 2 numbers multiplied equal -24 and 2 numbers added equals 10
Example:
a * -b = -24
a + -b = 10
Answer:
12 and -2
Step-by-step explanation:
Answer:
A = 12
-B = -2
12 x -2 = -24
12 + -2 = 10
hope this helps!
what are all the possible values of b that would make the following polynomial factorable? x^2+bx-8
Answer: -7, -2, 2, 7
The square root of 53
falls between ____
and _____
but it's closest to _____
(*Your answers must be between -20 and 20)
Answer:
Falls between 7 and 8
Step-by-step explanation:
Square root of 53 is 7.28
Answer:
7.280109
Step-by-step explanation:
The square root of 53 is 7.28
Please help! I have no clue how to do this problem. Any help would be appreciated:)
Answer:
13
Step-by-step explanation:
180-96=84
(4x-10)+(4x-10)= 84
X=13
Sherri lives in Canada and is considering buying a new sofa. If the price level in Canada falls and the price level in the United States does not change, Canadian manufactured sofas are relatively A) more expensive, so Sherri will likely purchase a U.S. manufactured sofa. B) more expensive, so Sherri will likely purchase a Canadian manufactured sofa. C) less expensive, so Sherri will likely purchase a U.S. manufactured sofa. D) less expensive, so Sherri will likely purchase a Canadian manufactured sofa. E) Both answers B and D could be correct depending on whether U.S. manufactured sofas were initially more expensive or less expensive than Canadian sofas.
If the price level in Canada falls and the price level in the United States does not change, Canadian-manufactured sofas are relatively less expensive, so Sherri will likely purchase a Canadian manufactured sofa.
This is because the decrease in price level in Canada would make Canadian goods more affordable, including Canadian manufactured sofas. This makes them a more attractive option for Sherri than U.S. manufactured sofas which would still be relatively more expensive even if their price level did not change. Answer D is correct in this scenario. However, if U.S. manufactured sofas were initially more expensive than Canadian sofas, then answer B could also be correct. Overall, Sherri's decision would depend on the initial price difference between Canadian and U.S. manufactured sofas, as well as her personal preferences and any other relevant factors.
Know more about price level here;
https://brainly.com/question/30264300
#SPJ11
Viola makes gift baskets for Valentine's Day. She plans to make 12 baskets each day. If there are 7 working days until the day she begins to sell the baskets, how many baskets will she have to sell?
Answer:84
Step-by-step explanation:
multiply the numbers
Answer:
84 baskets
Step-by-step explanation:
We know Viola plans to make 12 baskets each day to sell for Valentine's day. She is able to work for 7 days, giving her the oppertunity to make 84 baskets in the time period.
Here's the equation:
12 x 7 = 84
Viola can sell 84 baskets
Checked work:
84 ÷ 7 = 12
Ian has $6,000.00 to invest for 2 years. The table shows information about two investments Ian can make.
Ian makes no additional deposits or withdrawals. Which investment earns the greater amount of interest over a period of 2 years?
Investment X earns the greater amount of interest over a period of 2 years.
What is simple interest?Simple interest is a method of calculating interest on an amount for n period of time with a rate of interest of r. It is calculated with the help of the formula,
SI = PRT
where SI is the simple interest, P is the principal amount, R is the rate of interest, and T is the time period.
Let's consider that Ian invests in X, then:
Principle amount, P = $6,000
Time, T = 2 Years
Rate of Interest, R = 4.5% at simple Interest = 0.045
The interest earned is:
Interest = PRT = $6,000 × 0.045 × 2 = $540
Now, consider that Ian invests in Y, then:
Principle amount, P = $6,000
Time, n = 2 Years
Rate of Interest, R = 4% at Compound Interest = 0.04
The interest earned is:
Interest = P(1+R)ⁿ - P
= $6,000(1+0.04)² - $6,000
= $489.6
Since $540>$489.6, therefore, Investment X earns the greater amount of interest over a period of 2 years.
Learn more about Simple Interest here:
https://brainly.com/question/2793278
#SPJ1
GEOMETRY GEOMETRY GEOMETRY
____________________________________
In the figure below m∠PTR is known.
Which of the following relationships can be used to determine m∠RTQ?
The property of angles formed by a transversal line intersecting two parallel lines in order to determine the value of m∠RTQ in the given figure.
In this case, we can see that line PT is a transversal that intersects two parallel lines, RQ and AB. Therefore, we can use the property that alternate interior angles are congruent, which means that the angle formed by PT and RQ on one side will be equal to the angle formed by PT and AB on the other side.
Thus, we can write the relationship as:
m∠RTQ = m∠PTB
This means that the measure of angle RTQ is equal to the measure of angle PTB. Therefore, we can find the value of m∠RTQ by finding the value of m∠PTB, which can be calculated using the given value of m∠PTR and the fact that the angles in a triangle add up to 180 degrees.
In summary, to determine the value of m∠RTQ, we can use the relationship that alternate interior angles are congruent and set the measure of angle RTQ equal to the measure of angle PTB. We can then find the value of m∠PTB using the given information and use the fact that angles in a triangle add up to 180 degrees to calculate m∠RTQ.
To know more about Property of Angles visit:
https://brainly.com/question/12767194
#SPJ11
any luck with this question ??
Answer:
The difference is 10$
Step-by-step explanation:
tell me if you need an explanation
The position of an object moving along a line is given by the function s(t)=−4t^2+24t.
Find the average velocity of the object over the following intervals. (a) [1,10] (b) [1,9] (c) [1,8] (d) [1,1+h] where h>0 is any real number.
The position of an object moving along a line is given by the function s(t)=−4t²+24t, we need to find the average velocity of the object over the following intervals.(a) [1,10]The average velocity of the object is given by the formula: v_avg = (Δs) / (Δt)Δs = s(10) - s(1)= -4(10²) + 24(10) - (-4(1²) + 24(1))= -400 + 240 + 4 - 24= -180mΔt = 10 - 1= 9sSubstitute the values of Δs and Δt in the formula:v_avg = (Δs) / (Δt)=-180/9=-20m/s(b) [1,9]Δs = s(9) - s(1)= -4(9²) + 24(9) - (-4(1²) + 24(1))= -324 + 216 + 4 - 24= -128mΔt = 9 - 1= 8s
Substitute the values of Δs and Δt in the formula:v_avg = (Δs) / (Δt)=-128/8=-16m/s(c) [1,8]Δs = s(8) - s(1)= -4(8²) + 24(8) - (-4(1²) + 24(1))= -256 + 192 + 4 - 24= -84mΔt = 8 - 1= 7s
Substitute the values of Δs and Δt in the formula:v_avg = (Δs) / (Δt)=-84/7=-12m/s(d) [1, 1+h] where h > 0 is any real number.Δs = s(1 + h) - s(1)= -4(1 + h)² + 24(1 + h) - (-4(1²) + 24(1))= -4(1 + 2h + h²) + 24(1 + h) - 4 + 24= -4 - 8h - 4h² + 24 + 24h - 4 + 24= 40h - 4h²Δt = (1 + h) - 1= h
Substitute the values of Δs and Δt in the formula:v_avg = (Δs) / (Δt)=(40h - 4h²) / h= 40 - 4h
Explanation:From the above results, we can conclude that the average velocity of the object is -20 m/s for the interval [1,10], -16 m/s for the interval [1,9], -12 m/s for the interval [1,8], and 40 - 4h for the interval [1, 1+h].
To know more about average velocity visit:
brainly.com/question/28512079
#SPJ11
The product of -6 and a number
increased by 2 is -40. Find the number.
Let v1 span R3? Why or why not? and v3 . Does (1.V2.3) -4 -5 = -3 -9 6 Choose the correct answer below. A. Yes. Any vector in R3 except the zero vector can be written as a linear combination of these three vectors. B. No. The set of given vectors spans a plane in R3. Any of the three vectors can be written as a linear combination of the other two. O C. No. When the given vectors are written as the columns of a matrix A, A has a pivot position in only two rows. O D. Yes. When the given vectors are written as the columns of a matrix A, A has a pivot position in every ro row.
In this case, when the given vectors are written as the columns of a matrix A, A has a pivot position in only two rows. This means that the vectors are linearly dependent and do not span R3. Therefore, correct answer is C. No. When the given vectors are written as the columns of a matrix A, A has a pivot position in only two rows.
To determine if a set of vectors spans a given space, we need to check if they are linearly independent. If they are linearly independent, then they span the space. If they are linearly dependent, then they do not span the space.
One way to check for linear independence is to write the vectors as the columns of a matrix and reduce it to row echelon form. If there is a pivot position (a leading 1) in every row, then the vectors are linearly independent and span the space. If there is not a pivot position in every row, then the vectors are linearly dependent and do not span the space.
Therefore, the correct answer is C. No. When the given vectors are written as the columns of a matrix A, A has a pivot position in only two rows.
know more about matrix here
https://brainly.com/question/28180105#
#SPJ11
If the image of a and 3 under the function f (x) = x+ 3b are 5 and 6 respectively, fing the value of a and b
\(f(3)=3+3b=6 \implies \boxed{b=1} \\ \\ f(a) = a+3b = 5 \implies \boxed{a=1}\)
what does 234 to the power of 9 equal
find the general solution. express the solution in both vector and scalar form. x′= −6x 5y, y′= −5x 4y.
The general solution in both vector and scalar form is as follows:
Vector form:
\(\[\mathbf{x} = c_1\mathbf{v}_1e^{\lambda_1t} + c_2\mathbf{v}_2e^{\lambda_2t}\]\)
Scalar form:
\(\[\begin{aligned}x &= c_1v_{11}e^{\lambda_1t} + c_2v_{21}e^{\lambda_2t} \\y &= c_1v_{12}e^{\lambda_1t} + c_2v_{22}e^{\lambda_2t}\end{aligned}\]\)
What are eigenvalues?
Eigenvalues are a concept in linear algebra that are used to analyze the properties of linear transformations or matrices. In simple terms, eigenvalues represent the scaling factors by which a vector is stretched or compressed when it is transformed by a linear transformation or multiplied by a matrix.
To find the general solution for the given system of differential equations:
\(\[\begin{aligned}x' &= -6x + 5y \\y' &= -5x - 4y\end{aligned}\]\)
We can rewrite the system in matrix form as follows:
\(\[\mathbf{x'} = \begin{bmatrix} -6 & 5 \\ -5 & -4 \end{bmatrix} \mathbf{x}\]\)
where \($\mathbf{x} = \begin{bmatrix} x \\ y \end{bmatrix}$.\)
To find the general solution, we need to find the eigenvalues and eigenvectors of the coefficient matrix\($\begin{bmatrix} -6 & 5 \\ -5 & -4 \end{bmatrix}$.\)
The characteristic equation is given by:
\(\[\det(\mathbf{A} - \lambda \mathbf{I}) = 0\]\)
where \($\mathbf{A}$\) is the coefficient matrix,\($\lambda$\)is the eigenvalue, and \($\mathbf{I}$\)is the identity matrix.
Substituting the values, we have:
\(\[\begin{aligned}\det \begin{pmatrix} -6-\lambda & 5 \\ -5 & -4-\lambda \end{pmatrix} &= 0 \\(-6-\lambda)(-4-\lambda) - (5)(-5) &= 0 \\\lambda^2 + 10\lambda + 6 &= 0\end{aligned}\]\)
Solving this quadratic equation, we find the eigenvalues \(\lambda_1$ and $\lambda_2$.\left \{ {{y=2} \atop {x=2}} \right.\)
Once we have the eigenvalues, we can find the corresponding eigenvectors\($\mathbf{v}_1$ and $\mathbf{v}_2$\) by solving the equation:
\(\[(\mathbf{A}-\lambda\mathbf{I})\mathbf{v} = 0\]\)
Substituting the eigenvalues, we solve for the eigenvectors.
Let's denote the eigenvalues as \(\lambda_1$ and $\lambda_2$,\) and the corresponding eigenvectors as \(\mathbf{v}_1$ and $\mathbf{v}_2$.\)
Once we have the eigenvalues and eigenvectors, we can express the general solution in both vector and scalar form as follows:
Vector form:
\(\[\mathbf{x} = c_1\mathbf{v}_1e^{\lambda_1t} + c_2\mathbf{v}_2e^{\lambda_2t}\]\)
Scalar form:
\(\[\begin{aligned}x &= c_1v_{11}e^{\lambda_1t} + c_2v_{21}e^{\lambda_2t} \\y &= c_1v_{12}e^{\lambda_1t} + c_2v_{22}e^{\lambda_2t}\end{aligned}\]\)
wherec_1 andc_2 are arbitrary constants\(, $v_{ij}$\) that represent the \(i$-th\)component of the jth eigenvector and \($e^{\lambda_it}$\) represent the exponential term with the eigenvalue.
To learn more about Eigenvalues:
https://brainly.in/question/30757473
#SPJ4
Identify the vertex and y-intercept of the graph of the function y = (x + 2)^2 − 3. Use the drop-down menus to show your answer.
Answer:
Vertex of the quadratic function y = (x - h)² + k is:
point (h, k)Given function:
y = (x + 2)² − 3h = -2, k= -3Its vertex is (-2, -3)
The y-intercept is the point with x = 0:
y = (0 + 2)² − 3 = 4 - 3 = 1The y-intercept is (0, 1)
Answer:
Vertex: (-2, -3)
y intercept: 1
Step-by-step explanation:
In the diagram, △ABC∼△CDE. Write an equation for line k
Answer:
Step-by-step explanation:
Line AB is parallel to line CD (k) so they have the same slope.
slope=m=(y2-y1)/(x2-x1)
m=(6-0)/(4-0)
m=6/4=1.5
y=mx+b, we know m=1.5 so
y=1.5x+b, and we know it passes through point (4,0) so we can solve for b
0=1.5(4)+b
0=6+b
b=-6 so line k is
y=1.5x-6
h=(-5) answer needed now!!!!
Answer:
-2
Step-by-step explanation:
h(-5) represents the y value when x is -5.
Tell whether the set of ordered pairs {(1, 6), (2, 9), (3, 12), (4, 15)} satisfies a linear function. Explain.
A. No; there is no constant change in x that corresponds to a constant change in y.
B. Yes; there is no constant change in x that corresponds to a constant change in y.
C. No; there is a constant change in x that corresponds to a constant change in y.
D. Yes; there is a constant change in x that corresponds to a constant change in y.
Answer:
D
Step-by-step explanation:
If you put them into a graphing calculator they all align. Have a nice day.
suppose that two unrelated women who are having normal pregnancies, devondra and miranda, are given the first trimester screen. what is the probability that devondra gets a positive result and miranda gets a negative result? if 100 unrelated women with normal pregnancies are tested with the first trimester screen, what is the probability that at least 1 woman will receive a positive result?
The probability that at least one woman will receive a positive result if 100 unrelated women with normal pregnancies are tested with the first-trimester screen is 99.52%.
Suppose that two unrelated women, Devondra and Miranda, who are having normal pregnancies, are given the first-trimester screen.
This screen is a prenatal test that evaluates the risk of the fetus having certain chromosomal abnormalities, including Down syndrome.
The probability that Devondra gets a positive result and Miranda gets a negative result will be determined below.
For this purpose, the probability of getting a positive result on a first-trimester screen is 0.03, and the probability of getting a negative result is 0.97.
Therefore, the probability of Devondra getting a positive result and Miranda getting a negative result is:
P(Devondra gets a positive result AND Miranda gets a negative result) = P(Devondra gets a positive result) * P(Miranda gets a negative result)= 0.03 * 0.97
= 0.0291
= 2.91%.
Therefore, the probability that Devondra gets a positive result and Miranda gets a negative result is 2.91%.
Now, let's find out the probability that at least one woman will receive a positive result if 100 unrelated women with normal pregnancies are tested with the first-trimester screen.
The probability of getting a positive result is 0.03, and the probability of getting a negative result is 0.97.
We want to find the probability of at least one woman receiving a positive result.
It means that we will calculate the probability of no woman receiving a positive result and subtract it from 1.
Let's denote this probability by P(no woman receives a positive result).
P(no woman receives a positive result)
= P(first woman gets a negative result AND second woman gets a negative result AND ... AND hundredth woman gets a negative result)
= P(first woman gets a negative result) * P(second woman gets a negative result) * ... * P(hundredth woman gets a negative result) = 0.97 * 0.97 * ... * 0.97 (100 times)≈ 0.0048.
The probability that at least one woman will receive a positive result is:
P(at least 1 woman receives a positive result)= 1 - P(no woman receives a positive result)
= 1 - 0.0048
= 0.9952
= 99.52%.
For similar question on probability.
https://brainly.com/question/29276385
#SPJ11
3.8-5.5m-4n+8= Simplify
Answer:
-5.5m-4n+11.8
Step-by-step explanation:
what is the mathematicalrelationship between the number of behaviors and the information content (u) in bits? be precise and quantitative. you can provide a verbal statement or an equation.
The mathematical relationship between the number of behaviors and the information content (u) in bits can be described by the equation u = log2(N), where u represents the information content and N represents the number of behaviors.
The mathematical relationship between the number of behaviors and the information content can be quantitatively expressed using Shannon's information theory. According to Shannon's theory, the information content (u) in bits is directly related to the logarithm (base 2) of the number of possible behaviors (N).
The equation that represents this relationship is:
u = log2(N)
In this equation, "u" represents the information content in bits, and "N" represents the number of behaviors. The logarithm function measures the number of times 2 must be multiplied to get the number of behaviors.
For example, if there are 4 possible behaviors, then the information content would be:
u = log2(4)
u = log2(2^2)
u = 2 bits
This means that there are 2 bits of information content associated with 4 possible behaviors.
Learn more about Shannon's information theory
https://brainly.com/question/33335038
#SPJ11
could someone please help answer this, thank you, please no spam, will give brainliest, (homework help)!
Answer:
The answer would be W < - 18
L 4.6.3 Test (CST): Linear Equations
me.
OA. y+4= -3(x-3)
OB. y-4=-3(x+3)
OC. y-4=3(x+3)
OD. y+4=3(x-3)
(3,-4)
The correct option is OA. y+4= -3(x-3). L 4.6.3 Test (CST): Linear Equations Solution: We are given that a line passes through (3,-4) and has a slope of -3.
We will use point slope form of line to obtain the equation of liney - y1 = m(x - x1).
Plugging in the values, we get,y - (-4) = -3(x - 3).
Simplifying the above expression, we get y + 4 = -3x + 9y = -3x + 9 - 4y = -3x + 5y = -3x + 5.
This equation is in slope intercept form of line where slope is -3 and y-intercept is 5.The above equation is not matching with any of the options given.
Let's try to put the equation in standard form of line,ax + by = c=> 3x + y = 5
Multiplying all the terms by -1,-3x - y = -5
We observe that option (A) satisfies the above equation of line, therefore correct option is OA. y+4= -3(x-3).
Thus, the correct option is OA. y+4= -3(x-3).
For more question on equation
https://brainly.com/question/17145398
#SPJ8
Maria decorated her notebook with 4 rectangular stickers. Each sticker was 1/2 inches wide and 11/16 inches long
The total area of the stickers is (1/2) x (11/16) x 4 = 11/8 inches squared.
To calculate the total area of the stickers that were used to decorate the notebook, we need to know the dimensions of each sticker. In this case, the stickers were rectangular and each sticker was 1/2 inches wide and 11/16 inches long. To find the total area, we need to multiply the width of each sticker by the length of each sticker and then multiply this number by the number of stickers used. In this example, we have 4 stickers, so we must multiply (1/2) x (11/16) x 4. When this calculation is done, we end up with 11/8 inches squared as the total area of the stickers.
the complete question is :
Maria decorated her notebook with 4 rectangular stickers. Each sticker was 1/2 inches wide and 11/16 inches long
What was the area of each sticker?
learn more about area here
https://brainly.com/question/27683633
#SPJ4
\begin{tabular}{|rrl} \multicolumn{2}{|c}{ Taxable Income } & Tax Rate \\ \hline$0− & 9,950 & 10% \\ 9,950− & 40,525 & 12 \\ 40,525− & 86,375 & 22 \\ 86,375−164,925 & 24 \\ 164,925−209,425 & 32 \\ 209.425−523,600 & 35 \\ 525.600+ & & 37 \\ \hline \end{tabular} 5. Calculating Taxes Duela Dent is single and had $189,000 in taxable income. Using the rates from Table 2.3 in the chapter, calculate her income taxes. What is the average tax rate? What is the marginal tax rate?
Duela Dent's income taxes can be calculated using the given tax rates and her taxable income of $189,000. Her average tax rate and marginal tax rate can also be determined.
To calculate Duela Dent's income taxes, we need to determine the tax amount for each tax bracket that her taxable income falls into.
The taxable income of $189,000 falls into the following tax brackets:
$0-$9,950: Not applicable
$9,950-$40,525: ($40,525 - $9,950) * 12% = $3,546
$40,525-$86,375: ($86,375 - $40,525) * 22% = $9,992
$86,375-$164,925: ($164,925 - $86,375) * 24% = $19,110
$164,925-$209,425: ($189,000 - $164,925) * 32% = $7,744
$209,425-$523,600: Not applicable
$525,600 and above: Not applicable
Summing up the tax amounts for each bracket, Duela Dent's income taxes amount to $40,392.
The average tax rate is calculated by dividing the total tax amount ($40,392) by the taxable income ($189,000) and multiplying by 100:
Average Tax Rate = ($40,392 / $189,000) * 100 ≈ 21.37%
The marginal tax rate refers to the tax rate applied to an additional dollar of income. In this case, Duela Dent's marginal tax rate is 32%, which corresponds to the tax rate of the last bracket her taxable income falls into.
Learn more about income taxes here:
https://brainly.com/question/21595302
#SPJ11
HURRY PLEASE!!! What are the marginal
column totals?
The marginal column total for the given data is option D: 39; 16.
What is marginal total?
A set of marginal totals is produced when the cell frequencies of a (multidimensional) contingency table are added over one or more of the classifying variables. The row and column totals are the marginal totals for a two-dimensional table.
The sum of the row entries or the sum of the column entries are called the marginal totals. Marginal distributions are computed by dividing the row or column totals by the overall total.
The number of men who prefer to eat fish = 11
The number of women who prefer to eat fish = 6
The number of men who prefer to eat red meat = 28
The number of women who prefer to eat red meat = 10
So, the marginal column total for the number of men who prefer to eat fish and red meat collectively is -
11 + 28 = 39
The marginal column total for the number of women who prefer to eat fish and red meat collectively is -
6 + 10 = 16
Therefore, the marginal total is 39, 16.
To learn more about marginal total from the given link
https://brainly.com/question/13444663
#SPJ1